math1998

New Member
ارسال ها
336
لایک ها
224
امتیاز
0
#1
سلام دوستان با عذر خواهی و عرض معذرت جناب اقای math ولی همونطور که میدونید ماراتن های دیگه بلکل قدیمی شدن هیچکس دیگه نیگاهم بهشون نمیندازه این تایپیکم زدم تا به مباحث چندجمله ای و نامساوی بپردازیم و اندکی مواقع تابع برای امادگی م2 .
قوانین همون قوانین ماراتنای قبلی هرکس سوالی حل کرد سوال جدید میذاره .
1
ایا چندجمله ای غیر ثابت
با ضرایب صحیح وجود دارد که
همگی اعدادی اول باشند .

 

darya.f

New Member
ارسال ها
182
لایک ها
114
امتیاز
0
#2
پاسخ : ماراتن جبر

سلام دوستان با عذر خواهی و عرض معذرت جناب اقای math ولی همونطور که میدونید ماراتن های دیگه بلکل قدیمی شدن هیچکس دیگه نیگاهم بهشون نمیندازه این تایپیکم زدم تا به مباحث چندجمله ای و نامساوی بپردازیم و اندکی مواقع تابع برای امادگی م2 .
قوانین همون قوانین ماراتنای قبلی هرکس سوالی حل کرد سوال جدید میذاره .
1
ایا چندجمله ای غیر ثابت
با ضرایب صحیح وجود دارد که
همگی اعدادی اول باشند .


q عددى اول هست ...بنابر اىن چندجمله اى ثابت مىشه که تناقضه

---- دو نوشته به هم متصل شده است ----

سلام دوستان با عذر خواهی و عرض معذرت جناب اقای math ولی همونطور که میدونید ماراتن های دیگه بلکل قدیمی شدن هیچکس دیگه نیگاهم بهشون نمیندازه این تایپیکم زدم تا به مباحث چندجمله ای و نامساوی بپردازیم و اندکی مواقع تابع برای امادگی م2 .
قوانین همون قوانین ماراتنای قبلی هرکس سوالی حل کرد سوال جدید میذاره .
1
ایا چندجمله ای غیر ثابت
با ضرایب صحیح وجود دارد که
همگی اعدادی اول باشند .


q عددى اول هست ...بنابر اىن چندجمله اى ثابت مىشه که تناقضه

---- دو نوشته به هم متصل شده است ----

اگه مىشه خودتون سوال بزارىد من فعلا سوال خوب ندارم
 

math1998

New Member
ارسال ها
336
لایک ها
224
امتیاز
0
#3
پاسخ : ماراتن جبر


q عددى اول هست ...بنابر اىن چندجمله اى ثابت مىشه که تناقضه

---- دو نوشته به هم متصل شده است ----



q عددى اول هست ...بنابر اىن چندجمله اى ثابت مىشه که تناقضه

---- دو نوشته به هم متصل شده است ----

اگه مىشه خودتون سوال بزارىد من فعلا سوال خوب ندارم
به این سادگی ها هم نیست
ای که شما انتخاب کردی عددی خاصه و برای همه اعداد صحیح صادق نیست دقیق تر باشید.
 

darya.f

New Member
ارسال ها
182
لایک ها
114
امتیاز
0
#4
پاسخ : ماراتن جبر

به این سادگی ها هم نیست
ای که شما انتخاب کردی عددی خاصه و برای همه اعداد صحیح صادق نیست دقیق تر باشید.
ببخشىد مگه نباىد p ها متماىز باشن خب من الان ىه m,n اى پىدا کردم که p شون ىکى شد خب اىن تناقضه دىگه کارى به بقىه m,n ها نداره که!
 

Dadgarnia

New Member
ارسال ها
1,350
لایک ها
1,127
امتیاز
0
#5
پاسخ : ماراتن جبر

سلام دوستان با عذر خواهی و عرض معذرت جناب اقای math ولی همونطور که میدونید ماراتن های دیگه بلکل قدیمی شدن هیچکس دیگه نیگاهم بهشون نمیندازه این تایپیکم زدم تا به مباحث چندجمله ای و نامساوی بپردازیم و اندکی مواقع تابع برای امادگی م2 .
قوانین همون قوانین ماراتنای قبلی هرکس سوالی حل کرد سوال جدید میذاره .
1
ایا چندجمله ای غیر ثابت
با ضرایب صحیح وجود دارد که
همگی اعدادی اول باشند .

فرض كنيد
كه p يه عدد اوله و فرض كنيد
و داريم:

به ازاي هر kي طبيعي. پس k اي موجود است كه (p(m+kp عددي اول نيست.
 

math1998

New Member
ارسال ها
336
لایک ها
224
امتیاز
0
#6
پاسخ : ماراتن جبر

ببخشىد مگه نباىد p ها متماىز باشن خب من الان ىه m,n اى پىدا کردم که p شون ىکى شد خب اىن تناقضه دىگه کارى به بقىه m,n ها نداره که!
لزوما اینطور نیست چندجمله ای هم یه نوع تابعست اگر تابع به ازای 2 مقدار ثابت باشه یعنی در کل ثابته ؟؟!!!
 
آخرین ویرایش توسط مدیر

darya.f

New Member
ارسال ها
182
لایک ها
114
امتیاز
0
#7
پاسخ : ماراتن جبر

لزوما اینطور نیست چندجمله ای هم یه نوع تابعست اگر تابع به ازای 2 مقدار ثابت باشه یعنی در کل ثابته ؟؟!!!
نه کارى به ثابت بودن ىا نبودن ندارم ..بله در اون مورد شما درست مىگىد من منظورم اىنه که من 2تا m,n پىدا کردم که pشون ىکى شده در حالىکه براى هىچ m,nاى نباىد اىن جورى باشه نگفتم که چون واسه 2تا ىکى شده بره همه اىن جورىه!!
 

math1998

New Member
ارسال ها
336
لایک ها
224
امتیاز
0
#8
پاسخ : ماراتن جبر

نه کارى به ثابت بودن ىا نبودن ندارم ..بله در اون مورد شما درست مىگىد من منظورم اىنه که من 2تا m,n پىدا کردم که pشون ىکى شده در حالىکه براى هىچ m,nاى نباىد اىن جورى باشه نگفتم که چون واسه 2تا ىکى شده بره همه اىن جورىه!!
چرا نباید برا هیچ 2 تایی اینطور بشه ؟!
 

AHZolfaghari

Well-Known Member
ارسال ها
935
لایک ها
1,654
امتیاز
93
#9
پاسخ : ماراتن جبر

فکر کنم منظور خانوم darya.f این بوده باشه ! :






به همین ترتیب به ازای هر k طبیعی
برقرار است پس چند جمله ای
بی نهایت ریشه دارد که تناقضه پس باید ثابت باشه

پس
که q یه عدد اول هستش صدق میکنه


2
تمامی توابع f از اعداد گویای مثبت به خودش را بیابید بطوریکه :



همون قدر که چند جمله ای احتمال داره همون قدر نامساوی احتمال داره بیاد و همون قدر تابع احتمال داره بیاد پس چه خوبه رو هر سه قسمت کار بشه

ترجیحا هم از سوالات سبک شروع بشه و سطحش رفته رفته بالاتر بره​
 
آخرین ویرایش توسط مدیر

AhmadrezaBM

New Member
ارسال ها
213
لایک ها
40
امتیاز
0
#11
پاسخ : ماراتن جبر

ببخشید من سوال حل نکردم ولی یک سوال داشتم:
اگر a و b دو دنباله بصورت:

حاصل عبارت زیر را چطور میتوان بدست آورد؟




 

Dadgarnia

New Member
ارسال ها
1,350
لایک ها
1,127
امتیاز
0
#12
پاسخ : ماراتن جبر


2
تمامی توابع f از اعداد گویای مثبت به خودش را بیابید بطوریکه :



همون قدر که چند جمله ای احتمال داره همون قدر نامساوی احتمال داره بیاد و همون قدر تابع احتمال داره بیاد پس چه خوبه رو هر سه قسمت کار بشه

ترجیحا هم از سوالات سبک شروع بشه و سطحش رفته رفته بالاتر بره​
به راحتي مي توانيم بدست بياوريم:
با توجه به اين رابطه داريم:
 

math1998

New Member
ارسال ها
336
لایک ها
224
امتیاز
0
#13
پاسخ : ماراتن جبر

تمام توابع
را بیابید .


در ضمن منظور من این نبوذ که تابع کار نکنیم ولی گفتم که بیشتر اون دوتای دیگه رو کار کنید اگر هم سوال تابع میذارید سوالی ایده دار بذارید!!!
 

Dadgarnia

New Member
ارسال ها
1,350
لایک ها
1,127
امتیاز
0
#14
پاسخ : ماراتن جبر

تمام توابع
را بیابید .


در ضمن منظور من این نبوذ که تابع کار نکنیم ولی گفتم که بیشتر اون دوتای دیگه رو کار کنید اگر هم سوال تابع میذارید سوالی ایده دار بذارید!!!
فرض كنيد رابطه ي بالا
باشد پس داريم:
(1)

با استفاده از رابطه ي بالا داريم:


با استفاده از رابطه ي بالا و رابطه ي اصلي مسئله داريم:

با قرار دادن y=1 در رابطه ي بالا و استفاده از رابطه ي (1) داريم:

با استفاده از رابطه ي بالا داريم:

با قرار دادن x- در رابطه ي بالا داريم:

با استفاده از رابطه ي بالا داريم:

و (f(0 مي تونه صفر يا 1 يا 1- باشه كه نتيجه ميده سه تابع
و
و
در شرايط مسئله صدق مي كنند.
 

math1998

New Member
ارسال ها
336
لایک ها
224
امتیاز
0
#15
پاسخ : ماراتن جبر

فرض كنيد رابطه ي بالا
باشد پس داريم:
(1)

با استفاده از رابطه ي بالا داريم:


با استفاده از رابطه ي بالا و رابطه ي اصلي مسئله داريم:

با قرار دادن y=1 در رابطه ي بالا و استفاده از رابطه ي (1) داريم:

با استفاده از رابطه ي بالا داريم:

با قرار دادن x- در رابطه ي بالا داريم:

با استفاده از رابطه ي بالا داريم:

و (f(0 مي تونه صفر يا 1 يا 1- باشه كه نتيجه ميده سه تابع
و
و
در شرايط مسئله صدق مي كنند.
درسته فقط من یه ایده دیگه مد نظر دارم که که بعضی سوالا رو خیلی قشنگ حل میکنه اگر کسی چیز دیگه ای نگفت میگم .
 

Dadgarnia

New Member
ارسال ها
1,350
لایک ها
1,127
امتیاز
0
#16
پاسخ : ماراتن جبر

درسته فقط من یه ایده دیگه مد نظر دارم که که بعضی سوالا رو خیلی قشنگ حل میکنه اگر کسی چیز دیگه ای نگفت میگم .
يه راه ديگه به ذهنم رسيد:

پس داريم:



حالا در رابطه ي بالا
و
كه نتيجه مي دهد:
بقيه راه حل هم مشابه قبل است.
 

math1998

New Member
ارسال ها
336
لایک ها
224
امتیاز
0
#17
پاسخ : ماراتن جبر

يه راه ديگه به ذهنم رسيد:

پس داريم:



حالا در رابطه ي بالا
و
كه نتيجه مي دهد:
بقيه راه حل هم مشابه قبل است.
عالیه دقیقا همینه یه ایده ی باحال برا حل سوالات تابعیه اینم ایده ی مشابه البته این یکی خیلی ساده تره


 

Dadgarnia

New Member
ارسال ها
1,350
لایک ها
1,127
امتیاز
0
#18
پاسخ : ماراتن جبر

عالیه دقیقا همینه یه ایده ی باحال برا حل سوالات تابعیه اینم ایده ی مشابه البته این یکی خیلی ساده تره


فكر مي كنم يكي از شرايط اين سوال پيوسته بودن f باشه. اگه اينجوري باشه فرض كنيد رابطه ي بالا
باشه. داريم:

يكي از جواب ها
است در غير اين صورت تعريف مي كنيم
كه نتيجه مي دهد:



كه از توابع كوشي است. پس تابع ديگر
است كه C,k بزرگتر از صفرند.
 

Dadgarnia

New Member
ارسال ها
1,350
لایک ها
1,127
امتیاز
0
#19
پاسخ : ماراتن جبر

5
تمام توابع
و صعودي را بيابيد به طوري كه براي هر
داشته باشيم:


 

math1998

New Member
ارسال ها
336
لایک ها
224
امتیاز
0
#20
پاسخ : ماراتن جبر

5
تمام توابع
و صعودي را بيابيد به طوري كه براي هر
داشته باشيم:


سوال iran TST 2010 بوده فقط باید با توجه به اینکه پوشاست و مقدارگذاری مناسب میتونیم(راه حلش طولانیه) بذست بیاریم

و بعد میتونید نتیجه بگیرید
قسمت سختش فقط بدست اوردن
یادمه یه جایی برااین سوال 7-8 تا راه حل دیدم.:3:
 
بالا